Population Modeling using DE's

Click For Summary
Population modeling can be understood through differential equations, specifically the equation dP/dt = kP, where the population increases at a rate proportional to its current size. Given that a population doubles in 5 years, the constant k can be derived as k = ln(2)/5. To find the time required for the population to triple or quadruple, one can set up equations using the derived k value and solve for t by taking the natural logarithm of both sides. The initial population does not need to be assumed, as it cancels out in the equations. This approach simplifies the problem significantly, making it easier to find the desired timeframes.
prace
Messages
99
Reaction score
0
Hi,

I was wondering if anyone out here on a Friday night could help me understand population modeling. Here is what I have as a problem (this is pretty simple because my goal here is to understand the thinking behind the madness :rolleyes: )

-
The population of a certain community is known to increase at a rate proportional to the number of people present at any time. If the population has doubled in 5 years, how long will it take to triple, to quadruple?
-

So I understand that the rate = dy/dt and proportional translates to "something" = "some constant" times "something", or (dy/dt)=Ky, or in this case, dP/dt = kP.

Solving this DE I get the equation P(t)= P(initial)e^(kt). So all I am told is that the population doubles in 5 years. So what can I do? I can't assume an arbitrary number as the initial population can I? So I have one equation with three unknowns, P(initial), P(t), and k.

Any insight would be great. Thanks!
 
Physics news on Phys.org
you know that \frac{dP}{dt} = kP. The solution is:

P(t) = P_{0}e^{kt}

You also know that 2P_{0}= P_{0}e^{5k}. What can you do from here? Solve for k:

e^{5k} = 2

k = \frac{\ln 2}{5}.
Then solve the equations for t:

3P_{0} = P_{0}e^{(\frac{\ln 2}{5})t}
4P_{0} = P_{0}e^{(\frac{\ln 2}{5})t}
 
Last edited:
Oh man... So here the P_{0}'s will cancel and all I have to do is take the natural logs of both sides to get t? man... I think I was making it a lot harder than it really was. Thank you for taking the time to show me this.
 
Question: A clock's minute hand has length 4 and its hour hand has length 3. What is the distance between the tips at the moment when it is increasing most rapidly?(Putnam Exam Question) Answer: Making assumption that both the hands moves at constant angular velocities, the answer is ## \sqrt{7} .## But don't you think this assumption is somewhat doubtful and wrong?

Similar threads

  • · Replies 12 ·
Replies
12
Views
2K
  • · Replies 1 ·
Replies
1
Views
2K
Replies
13
Views
3K
  • · Replies 5 ·
Replies
5
Views
2K
  • · Replies 10 ·
Replies
10
Views
6K
Replies
5
Views
2K
  • · Replies 7 ·
Replies
7
Views
2K
Replies
17
Views
4K
  • · Replies 73 ·
3
Replies
73
Views
10K
  • · Replies 5 ·
Replies
5
Views
4K